4
$\begingroup$

The sequence of colossally abundant (CA) numbers, $a(n)$ (OEIS A004490), consists of positive integers that maximize the ratio $\frac{\sigma(m)}{m^{1+\epsilon}}$ for some $\epsilon > 0$.

A known property concerning their long-term divisibility is the following:

Theorem: For every fixed integer $k \ge 1$, there exists an index $A$ (depending on $k$) such that $k$ divides $a(n)$ for all $n > A$.

I am looking for a reference or a formal proof of this theorem.

The intuition is clear: since the prime exponents in the factorization of $a(n)$ grow indefinitely, they must eventually satisfy the required exponents for any fixed $k$. However, I am seeking a formall proof, ideally from the works of Ramanujan, Alaoglu, or Erdős.

$\endgroup$
3
  • 5
    $\begingroup$ this is actually true for super-abundant numbers of which CA numbers are a subset - a fairly straitghtforward proof is given in Broughan Equivalents of RH Volume I on page $150$ $\endgroup$ Commented Sep 30 at 21:28
  • $\begingroup$ Thank you very much, do you have the link or do you know where I can get it?, I can't find it $\endgroup$ Commented Oct 1 at 3:31
  • 3
    $\begingroup$ you can get the proof from the two original papers - result is Proposition 1 SA2 in Caveney, Nicolas, Sondow arxiv.org/abs/1110.5078 which uses a few properties of SA numbers proved by Alaoglu and Erdos in the paper available at users.renyi.hu/~p_erdos/1944-03.pdf $\endgroup$ Commented Oct 1 at 4:06

1 Answer 1

4
$\begingroup$

Here is a short and self-contained proof.

Fix an integer $k\ge 1$. It suffices to show that for each prime power $p^e$ dividing $k$ there is an index $A_{p,e}$ so that every CA number $m$ with index $>A_{p,e}$ satisfies $v_{p}(m)\ge e$. Taking $A=\max_{p^e|k}A_{p,e}$ gives the theorem.

Recall the defining property: $m$ is CA iff there exists $\varepsilon > 0$ such that $$ F(n,\varepsilon):=\frac{\sigma(n)}{n^{1+\varepsilon}} $$ is maximized at $n=m$. For each CA number $m$ choose one such $\varepsilon = \varepsilon_m>0$.

We want to show that $\varepsilon_m \to 0$ as $m\to\infty$. For fixed $\varepsilon_0 >0$, we have $$ F(n,\varepsilon_0)=\frac{\sigma(n)}{n}\cdot n^{-\varepsilon_0}\le \tau(n)\,n^{-\varepsilon_0} $$ and since $\tau(n)=n^{o(1)}$, we have $$\tau(n)\, n^{-\varepsilon_0} \xrightarrow{n\to\infty} 0$$ and hence $F(\cdot,\varepsilon_0)$ achieves its global maximum at some finite $n$, so only finitely many CA numbers can arise as maximizers for $\varepsilon\ge\varepsilon_0$. Since this holds for every $\varepsilon_0 > 0$, the chosen $\varepsilon_m$ for CA numbers must tend to $0$ along any subsequence with $m\to\infty$.

Now, fix a prime $p$ and an exponent $e\ge 1$. Assume, toward a contradiction, that there are infinitely many CA numbers $m$ with $v_p(m) = v < e$. For such an $m$ let $\varepsilon=\varepsilon_m$ (so $\varepsilon$ can be taken arbitrarily small along the infinite family). Consider $$ m':=m\cdot p^{e-v} $$ so that $v_p(m') = e$. Also, we have $$ \frac{F(m',\varepsilon)}{F(m,\varepsilon)} = \frac{\sigma(p^{e})/p^{e(1+\varepsilon)}}{\sigma(p^{v})/p^{v(1+\varepsilon)}} = \frac{\sigma(p^{e})}{\sigma(p^{v})}\,p^{-(e-v)(1+\varepsilon)} $$ and hence $$ \lim_{\varepsilon\to 0}\frac{F(m',\varepsilon)}{F(m,\varepsilon)} = \frac{p^{e+1}-1}{p^{v+1}-1}\,p^{-(e-v)} $$ and $$ \frac{p^{e+1}-1}{p^{v+1}-1} - p^{e-v} = \frac{p^{e+1}-1 - p^{e-v}(p^{v+1}-1)}{p^{v+1}-1} = \frac{p^{e-v}-1}{p^{v+1}-1}>0 $$ implying $$\frac{\sigma(p^{e})}{\sigma(p^{v})}\,p^{-(e-v)}>1$$ and hence, for $\varepsilon$ sufficiently small, we have $F(m',\varepsilon) >F(m,\varepsilon)$. This contradicts the fact that $m$ maximizes $F(\cdot,\varepsilon)$. Therefore only finitely many CA numbers can have $v_p<e$ and equivalently, for all sufficiently large CA indices we have $v_p\ge e$.

Putting these finite bounds together for every prime power dividing $k$ proves: there is $A$ so that for every CA number $a(n)$ with $n>A$ we have $k\mid a(n)$. This completes the proof.

$\endgroup$

You must log in to answer this question.

Start asking to get answers

Find the answer to your question by asking.

Ask question

Explore related questions

See similar questions with these tags.